You are on page 1of 14

myweb.ttu.

edu/bban
lev.ban@ttu.edu

Real Analysis
Byeong Ho Ban
Mathematics and Statistics
Texas Tech University

Chapter 1. Measure

1.
Proof. 

2.
Proof. 

3. Let M be an infinite σ - algebra.


(a) M contains an infinite sequence of disjoint sets.
(b) card(M) ≥ c
Proof. Solution for (a).
If the disjoint sets can be empty set, then {Ei }∞ 1 where Ei = ∅ ∀i ∈ N is the infinite sequence that we
were finding, so it is obvious.
Thus, let’s prove that M has an infinite sequence of nonempty disjoint sets. By way of contradiction,
n
assume that there exists no such Sn sequence.
S thereSnexists aScollection of sets {Ei }1 ⊂ M such that
Then
n = max k ∈ N|Ei ∈ M and S 1 Ei = S K∈M K and 1 Ei = K∈M K, where Ei ∩ Ej = ∅ if i 6= j. And
let {Qi }n1 ⊂ M be satisfying n1 Qi = K∈M K. Then M should be the σ−algebra generated by {Qi }n1 .
Sk let
Otherwise, there exists Sm R ∈ M such that R 6
= 1 Qi = Uk ∀k ∈ {1, 2, . . . , n}. Then, ∃m ∈ {1, 2, . . . , n}
such that Qk 6⊂ R/ 1 Qi = Um 6= ∅ ∀k ∈ {1, 2, . . . , n}. But note that Um ∈ M and it means Um ⊂ Qk
c
for some k ∈ {1, 2, . . . , n}. So we can divide
S Qk into Um ∩ Qk and Um ∩ Qk . Thus, we have n+1 nonempty
disjoint subsets which is union of them is K∈M K. And it contradicts to the definition of n.
Note that the σ−algebraS generated by {Qi }n1 has at most 2n elements because of the following argument.
Note that ∀Q ∈ M Q = Qi ⊂Q Qi , so ∀i ∈ N Q ∩ Qi = ∅ or Qi ⊂ Q.(∵ Qi 6= Qj = ∅ if i 6= j)
Now define a function κ : P({1, 2, · · · , n}) −→ M({Qi }∞
S
1 ) such that κ(I) = i∈I Qi .
Observe that the map κ is bijection. S
First of all, it is injection by following argument. If I 6= J, then ∃i ∈ I/J, so Qi 6⊂ j∈J Qj = κ(J). Thus,
Qi ⊂ κ(I) 6= κ(J). So, κ is injection.
And it is surjection because ∀Q ∈ M({Qi }∞
S
1 ), Q = Qi ⊂Q Qi .
However, it means M is finite which is contradiction to our assumption. Therefore, M should contain an
infinite sequence of disjoint sets.

Solution for (b)


Let’s define a function f : P(N) −→ M. S From (a), we know that the M contains an infinite sequence of
nonempty disjoint setsS {Ai }. Let f (I) = i∈I Ai . Then f is an injection. That is because if I 6= J , then
∃i ∈ I/J, then Ai 6⊂ j∈J Aj = f (J), so Ai ⊂ f (I) 6= f (J). Therefore, c = card(P(N)) ≤ card(M).
1
2

4. An algebra A is a σ−algebra iff A is closedSunder countable increasing unions


(i.e., if {Ei }∞
1 ⊂ A and E1 ⊂ E2 ⊂ · · · , then

1 Ej ∈ A )

Proof. (−→)
Suppose that A is a σ−algebra and S∞that {Ei }∞
1 ⊂ A where E1 ⊂ E2 ⊂ · · · .
Then, by definition of σ−algebra, 1 Ei ∈ A.
(←−)
Suppose that ∞ ∞
S
1 Ei ∈ A whenever {Ei }1 ⊂ A and E1 ⊂ E2 ⊂ · · · . Since A is an algebra, A is closed
under complement. So it suffices to prove that A is closed under countable union. Suppose that {Fi }∞
1 is
S2 S3 Sk Sk
a sequence in A. Observe that F1 ⊂ 1 Fi ⊂ 1 Fi ⊂ · · · ⊂ 1 Fi ⊂ · · · . And note that 1 Fi ∈ A since
A is algebra so closed under finite union.
Thus, since A is closed under countable increasing unions, ∞
S Sk S∞
1 1 Fi = 1 Fi ∈ A. Therefore, A is closed
under countable union so is σ−algebra. 

5. If M is the σ−algebra generated by E, then M is the union of the σ−algebras generated by F as F


ranges over all countable subsets of E.
(Hint: Show that the latter is a σ−algebra.)
S
Proof. Following the given hint, let’s show that S = F ⊂E(F is countable) M(F) is a σ−algebra.
Suppose that E ∈ A, then
S ∃F ⊂ E such that F is countable and E ∈ M(F). Since M(F) is a σ−algebra,
c c
E ∈ M(F), so E ∈ F ⊂E(F is countable) M(F) = S.(Closed under complement)

Let {Ei }∞1 ⊂ S. Note that, ∀i ∈ N ∃Fi ⊂ E such that each Fi are countable and Ei ∈ Fi . From
set theory, we know that
V= ∞
S
1 Fi

is also countable. Thus, M(V) ⊂ S. Since 1 Ei ∈ M(V), it is true that ∞


S∞ S
1 Ei ∈ S.
(Closed under countable union)
Thus, S is a σ−algebra.

It is obvious that S ⊂ M since each M(F) are subsets of M. Now, observe that E ⊂ S because for
any E ∈ E, {E} is a countable set, so M({E}) ⊂ S and E should be in S. Thus, E ⊂ S ⊂ M. Therefore,
S = M(S) = M(E) = M.
∴S=M 

6.
Proof. 

7.
Proof. 

8.
Proof. 
3

9. If (X, M, µ) is a measure space and E, F ∈ M, then µ(E) + µ(F ) = µ(E ∪ F ) + µ(E ∩ F ).


Proof. Observe that E/F, E ∪ F, E ∩ F ∈ M since M is a σ−algebra.
And note that, by countable additivity,
µ(E) + µ(F ) = µ(E/F ) + µ(E ∩ F ) + µ(F ) = {µ(E/F ) + µ(F )} + µ(E ∩ F ) = µ(E ∪ F ) + µ(E ∩ F )
∵ E/F ∩ (E ∩ F ) = E/F ∩ F = ∅. 

10. Given a measure space (X, M, µ) and E ∈ M, define µE (A) = µ(A ∩ E) for A ∈ M. Then µE is a
measure.
Proof. In order to prove that µE is a measure, we need to show that µE satisfies the following two properties.

(1) µE (∅) = 0
S∞
(2) µE ( 1 Ei ) = Σ ∞
1 µE (Ei ) where Ei 6= Ej if i 6= j

The first property is obvious because


µE (∅) = µ(E ∩ ∅) = µ(∅) = 0 (∵ µ is a measure).
Let’s show that µE satisfies the second properties.
Let {Ei }∞
1 ⊂ M be a sequence of disjoint sets in M. Then, observe that

µE ( ∞
S S∞ S∞
1 Ei ) = µ(E ∩ 1 Ei ) = µ( 1 E ∩ Ei )

Here, we know that {E ∩ Ei }∞


1 is a sequence of disjoints sets in M. Since µ is a measure, by countable
additivity,
µ( ∞ ∞ ∞
S
1 A ∩ Ei ) = Σ1 µ(A ∩ Ei ) = Σ1 µE (E)

Therefore,
µE ( ∞ ∞
S
1 Ei ) = Σ1 µE (E)

Thus, µE is a measure. 

11.
Proof. 

12.
Proof. 

13.
Proof. 

14.
Proof. 
4

15.
Proof. 

16.
Proof. 

17. If µ∗ is an outer measure on X, and {Aj }∞ ∗


1 is a sequence of disjoint µ −measurable sets, then
µ∗ (E ∩ ( ∞ ∞ ∗
S
1 Aj ))=Σ1 µ (E ∩ Aj )
for any E ⊂ X.
Proof. Let E ⊂ X be given. Note that, by subadditivity,

! ∞
!
[ [
µ∗ E ∩ ( A j ) = µ∗ (E ∩ Aj ) ≤ Σ∞ ∗
1 µ (E ∩ Aj )
1 1

Thus, in order to prove the statement, we need to show the reverse inequality.
Reminding that each Aj are µ∗ −measurable, for any n ∈ N observe the below.
n
! n
! n
!
[ [ [
µ∗ E ∩ ( Aj ) = µ∗ E ∩ ( Aj ) ∩ A1 + µ∗ E ∩ ( Aj ) ∩ Ac1
1 1 1
n
!
[
= µ∗ (E ∩ A1 ) + µ∗ E ∩ ( Aj )
2
n
! n
!
[ [
= µ∗ (E ∩ A1 ) + µ∗ E ∩ ( Aj ) ∩ A2 + µ∗ E ∩ ( Aj ) ∩ Ac2
2 2
n
!
[
= Σ21 µ∗ (E ∩ Aj ) + µ∗ E ∩ ( Aj )
3
= · · · = Σn1 µ∗ (E ∩ Aj )
Due to monotonicity which is a property of outer measure,

! n
!
[ [
µ∗ E ∩ ( Aj ) ≥ µ∗ E ∩ (Aj ) ≥ Σn1 µ∗ (E ∩ Aj )
1 1

Thus, as n goes to infinity,



!
[
µ∗ E ∩ ( Aj ) ≥ Σ∞ ∗
1 µ (E ∩ Aj )
1

18. Let A ⊂ P(X) be an algebra, Aσ the collection of countable unions of sets in A, and Aσδ the collec-
tion of countable intersection of sets in Aσ . Let µ0 be a premeasure on A and µ∗ the induced outer measure.

a. For any E ⊂ X and  > 0, there exists A ∈ Aσ with E ⊂ A and µ∗ (A) ≤ µ∗ (E) + 
b. If µ∗ (E) < ∞, then E is µ∗ − measurable iff there exists B ∈ Aσδ with E ⊂ B and µ∗ (B \ E) = 0.
c. If µ0 is σ-finite, the restriction µ∗ (E) < ∞ in (b) is superfluous.
5

Proof. Proof for (a) .


Let E ⊂ X and  > 0 are given. Note that
( ∞
)
[
µ∗ (E) = inf Σ∞
1 µ0 (Aj ) | Aj ∈ A, E ⊂ Aj
1

Since µ is an outer measure induced from given µ0 .
By definition of infimum, there exists a sequence of sets {Aj }∞
1 ⊂ A such that

E ⊂ j∈N Aj and Σj∈N µ0 (Aj ) ≤ µ∗ (E) + .


S

Here, let A = j∈N Aj , then A ∈ Aσ and by the property of infimum, µ∗ (A) ≤ Σj∈N µ0 (Aj ), so
S

µ∗ (A) ≤ µ∗ (E) + 
Therefore, the A is the set we were finding.

Proof for (b)


( =⇒ ) Suppose that µ∗ (E) < ∞ and E is µ∗ −measurable. By the consequence of part (a), for each n ∈ N,
we can choose Bn ∈ Aσ that satisfies the inequality below.
1
µ∗ (Bn ) ≤ µ∗ (E) +
n
T
Let B = n∈N Bn , then B ∈ Aσδ and by monotonocity,
1
µ∗ (B) ≤ µ∗ (Bn ) ≤ µ∗ (E) +
n
If we take a limit as n → ∞, µ∗ (B) ≤ µT

(E).
Since E ⊂ Bn for any n ∈ N, E ⊂ B = n∈N Bn . So, by monotonicity, µ∗ (E) ≤ µ∗ (B)
Therefore, µ∗ (E) = µ∗ (B). Remember that E is µ∗ −measurable. So,
µ∗ (B) = µ∗ (B ∩ E) + µ∗ (B ∩ E c )
= µ∗ (E) + µ∗ (B \ E) (∵ E ⊂ B)
Since µ∗ (E) < ∞,
µ∗ (B \ E) = µ∗ (B) − µ∗ (E) = 0

( ⇐= ) Suppose that there exists B ∈ Aσδ with E ⊂ B and µ∗ (B \ E) = 0. Observe that Aσδ ⊂
M(A). Since, by a theorem in our book, M(A) is s subset of a collection of all µ∗ −measurable sets, B is
µ∗ −measurable. Also, by Caratheodory’s Theorem, B \ E is also µ∗ −measurable. Thus,
B \ (B \ E) = B ∩ (B ∩ E c )c = B ∩ (B c ∪ E) = B ∩ E = E
is also µ∗ −measurable.
Proof for (c).
( =⇒ ).
Suppose that µ0 is σ−finite and E is µ∗ −measurable. Thus, X = n∈N En where µ∗ (En ) = µ0 (En ) < ∞
S
∀n ∈ N
By part (a), we can choose a sequence {Aij }i,j∈N ⊂ Aσ such that (En ∩ E) ⊂ Ank and µ∗ (En ∩ E) ≤
µ∗ (Ank ) < µ∗ (En ∩ E) + k21n ∀n, k ∈ N
Here, we know that µ∗ (En ∩ E) ≤ µ∗ (En ) < ∞ ∀n ∈ N, thus,
1
µ∗ (Ank \ (En ∩ E)) = µ∗ (Ank ) − µ∗ (En ∩ E) <
k2n
6
S S
Let Ak = n∈N Ank , then Ak ∈ Aσ and E ⊂ Ak . Now, note that Ak \ E ⊂ n∈N (Ank \ (E ∩ En )). Thus,
observe that
1
µ∗ (Ak \ E) ≤ Σn∈N µ∗ (Ank \ (E ∩ En )) <
k

T
Let B = k∈N Ak , then B ∈ Aσδ and E ⊂ B. Observe that
1
µ∗ (B \ E) ≤ µ∗ (Ak \ E) <
k
Therefore, if we send k to ∞, we get µ∗ (B \ E) = 0.

( ⇐= ).
Note that we have not used the condition either ”µ∗ (E) < ∞” or ”µ0 is a σ−finite” in the proof of part
(b). Thus, our proof in the part (b) is still valid. Below is just the copy of the proof.

Suppose that there exists B ∈ Aσδ with E ⊂ B and µ∗ (B \ E) = 0. Observe that Aσδ ⊂ M(A). Since, by
a theorem in our book, M(A) is s subset of a collection of all µ∗ −measurable sets, B is µ∗ −measurable.
Also, by Caratheodoy’s Theorem, B \ E is also µ∗ −measurable. Thus,

B \ (B \ E) = B ∩ (B ∩ E c )c = B ∩ (B c ∪ E) = B ∩ E = E

is also µ∗ −measurable. 

19. Let µ∗ be an outer measure on X induced from a finite premeasure µ0 . If E ⊂ X, define the
inner measure of E to be µ∗ (E) = µ0 (X) − µ∗ (E c ). Then E is µ∗ −measurable iff µ∗ (E) = µ∗ (E)
Proof. Note that µ∗ is finite since µ0 (X) = µ∗ (X).
( =⇒ )
Suppose that E is µ∗ −measurable. Then

µ∗ (X) = µ∗ (X ∩ E) + µ∗ (X ∩ E c )
= µ∗ (E) + µ∗ (E c )

Since µ∗ (E c ) < ∞

µ∗ (E) = µ∗ (X) − µ∗ (E c ) = µ0 (X) − µ∗ (E c ) = µ∗ (E)

( ⇐= )
Suppose that µ∗ (E) = µ∗ (E).
By part (a) of Homework question 2, for each n ∈ N, we can choose Bn ∈ Aσ with E ⊂ Bn such that
1
µ∗ (Bn ) ≤ µ∗ (E) +
n
T
Let B = n∈N Bn , then B ∈ Aσδ and by monotonocity,
1
µ∗ (B) ≤ µ∗ (Bn ) ≤ µ∗ (E) +
n
If we take a limit as n → ∞, µ∗ (B) ≤ µT

(E).
Since E ⊂ Bn for any n ∈ N, E ⊂ B = n∈N Bn . So, by monotonicity, µ∗ (E) ≤ µ∗ (B)
Therefore, µ∗ (E) = µ∗ (B).
7

Note that Aσδ is a subset of the collection of all µ∗ −measurable sets. So B is µ∗ −measurable. Thus,
µ∗ (X) = µ∗ (X ∩ B) + µ∗ (X ∩ B c )
= µ∗ (B) + µ∗ (B c )
= µ∗ (E) + µ∗ (B c )
= µ∗ (E) + µ∗ (B c )
= µ∗ (X) − µ∗ (E c ) + µ∗ (B c )
Since µ∗ is finite,
µ∗ (E c ) = µ∗ (B c )
Observe that, since B is µ∗ −measurable,
µ∗ (E c ) = µ∗ (E c ∩ B) + µ∗ (E c ∩ B c )
= µ∗ (B \ E) + µ∗ (B c ) (∵ B c ⊂ E c )
= µ∗ (B \ E) + µ∗ (E c )
Since µ∗ (E c ) < ∞, µ∗ (B \ E) = 0.
By part (b) of homework question 2, since we found the set B, it is true that E is µ∗ −measurable. 

20.
Proof. 

21.
Proof. 

22.
Proof. 

23.
Proof. 

24. Let µ be a finite measure on (X, M), and let µ∗ be the outer measure induced by µ. Suppose that
E ⊂ X satisfies µ∗ (E) = µ∗ (X) (but not that E ∈ M).

a. If A, B ∈ M and A ∩ E = B ∩ E, then µ(A) = µ(B).


b. Let ME = {A ∩ E|A ∈ M}, and define the function ν on ME defined by ν(A ∩ E) = µ(A)(which
makes sense by (a)). Then ME is a σ−algebra on E and ν is a measure on ME
Proof. Note that µ∗ is a finite outer measure.
Proof for a
Suppose that A, B ∈ M and that A ∩ E = B ∩ E.
Observe that, since µ∗ is a measure on M and any sets in M are µ∗ −measurable,
µ∗ (X) = µ∗ (E) = µ∗ (E ∩ B) + µ∗ (E ∩ B c ) = µ∗ (E ∩ B) + µ∗ (E ∪ B) − µ∗ (B)
µ∗ (X) = µ∗ (E) = µ∗ (E ∩ A) + µ∗ (E ∩ Ac ) = µ∗ (E ∩ A) + µ∗ (E ∪ A) − µ∗ (A)
8

Since E ⊂ E ∪ A ⊂ X and E ⊂ E ∪ B ⊂ X, by monotonicity,

µ∗ (E ∪ B) = µ∗ (E ∪ A) = µ∗ (X)

Summing up, observe that

µ∗ (X) = µ∗ (E ∩ B) + µ∗ (E ∪ B) − µ∗ (B)
= µ∗ (E ∩ A) + µ∗ (E ∪ A) − µ∗ (B)
= µ∗ (X) + µ∗ (A) − µ∗ (B)

Thus, µ(A) = µ∗ (A) = µ∗ (B) = µ(B).

Proof for b
Firstly, ME is closed under complement.(Set subtraction from E)
Let A ∩ E ∈ ME , and observe that

E \ (A ∩ E) = E ∩ (A ∩ E)c
= E ∩ (Ac ∪ E c ) = Ac ∩ E

Since Ac ∈ M, Ac ∩ E ∈ ME , so E \ (A ∩ E) ∈ ME
Secondly, it is closed under countable union.
Let {An }n∈N ⊂ M, then {An ∩ E}n∈N is a sequence of sets in ME . Now, observe that
!
[ [ [
(An ∩ E) = An ∩ E ∈ ME (∵ An ∈ M)
n∈N n∈N n∈N

Thus, it is closed under countable unions.


Therefore, ME is a σ−algebra.

Let’s prove that ν is a measure on ME .


Let A ∩ E = B ∩ E and A ∩ E, B ∩ E ∈ ME , then by part (a), we know that ν(A ∩ E) = µ(A) = µ(B) =
ν(B ∩ E). So ν is well defined.
Now, observe that

ν(∅) = ν(∅ ∩ E) = µ(∅) = 0

We still need to prove the countable additivity. Let {An ∩ E}n∈N ⊂ ME be a sequence of disjoint sets.
S that{An }n∈N ⊂ M is not a sequence of disjoint sets.
However, it is possible
n−1
So let Bn = An \ j=1 Aj ∈ M.
Observe the below
n−1
!
\
Bn ∩ E = An ∩ E ∩ Acj
j=1
n−1
!
\
= (An ∩ E) ∩ (Acj ∩ E)
j=1

= An ∩ E (∵ (An ∩ E) ⊂ (Acj ∩ E) if n 6= j)
∴ An ∩ E = Bn ∩ E ∀n ∈ N

Now, with the given information, observe that


9

!
[
Σn∈N ν(An ∩ E) = Σn∈N ν(Bn ∩ E) = Σn∈N µ(Bn ) = µ Bn
n∈N
!! !! !
[ [ [
=ν E∩ Bn =ν E∩ An =ν An ∩ E
n∈N n∈N n∈N

Therefore, ν is satisfies countable additivity, so it is a measure. 

25.
Proof. 

26. Prove Proposition 1.20. (Use Theorem 1.18)


Proposition 1.20.
If E ∈ Mµ and µ(E) < ∞, then for every  > 0 there is a set A that is a finite union of open intervals
such that µ(E 4 A) < 
Proof. Let E ∈ Mµ , µ(E) < ∞, and  > 0. Then, by Theorem 1.18, there exists an open set such that

E ⊂ U , and µ(U
S ) < µ(E)+ 2 . Since every open set is an countable unions of disjoint open interval {Ik }k∈N ,
we have U = k∈N Ik . Since µ is a measure, we know that
[
µ(U ) = µ( Ik ) = Σk∈N µ(Ik )
k∈N

Since µ(U ) < µ(E) + and µ(E) is finite, the series is convergent.
2
It means there exists N ∈ N such that

Σ∞k=N +1 µ(Ik ) <
2
SN
Let A = k=1 Ik . Observe that

µ(E \ A) ≤ µ(U \ A) = Σ∞
k=N +1 µ(Ik ) <
2

µ(A \ E) ≤ µ(U \ E) = µ(U ) − µ(E) <
2
Thus,
 
µ(A 4 E) = µ(E \ A) + µ(A \ E) < + =
2 2
SN
∴ µ(A 4 E) <  ( With A = k=1 Ik )

27. Prove Proposition 1.22a. (Show that if x, y ∈ C and x < y, there exists z 6∈ C such that x < z < y)
Proposition 1.22a
C is compact, nowhere dense, and totally disconnected(i.e., the only connected subsets of C are single
points). Moreover, C has no isolated points.
Proof. .

1. C is compact.
(→) Note that C ⊂ [0, 1]. Also, note that C c is open since it is a countable disjoint unions of open intervals,
so C is closed. Since C is closed and bounded in R, C is compact.
10

2. C is totally disconnected.
(→) Let x, y ∈ C and x < y. By way of contradiction, assume that ∀z ∈ (x, y), z ∈ C. Note that
(x, y) ⊂ C . Also, note that y − x > 0 since x 6= y. Thus, by monotonicity, m(C) ≥ y − x > 0. But it is
contradiction to Proposition 1.22b. Thus, there should exist z ∈ (x, y) such that z 6∈ C

3. (This Proof is not correct) C has no isolated point.


(→) Let x ∈ C, then x = 3pk for some integers k and p where 0 ≤ p ≤ 3k and gcd(3, p) = 1.
j −1
Let  > 0 be given. Observe that for some integer j, if y = p33j+k , then y ∈ C (since gcd(p3j − 1, 3) = 1 and
j j
0 ≤ p3j −1 ≤ 3j+k ) and |x−y| = | p3 3−1−p3
j+k
1
| = | 3j+k 1
|. And we can choose j ∈ N such that | 3k+j | < . There-
fore, for arbitrary positive real number  > 0 and any x ∈ C, there exists y ∈ C such that y ∈ (x − , x + ).
So there is no isolated point in C.

4. C is nowhere dense.
(→) Let x ∈ C and  > 0. Since C has no isolated point, ∃y ∈ C such that y ∈ (x − , x + ). Also
observe that, since C is totally disconnected, ∃z ∈ (x, y) ⊂ (x − , x + ) such that z 6∈ C. Note that
(x, y) ⊂ (x − , x + ) and (x, y) 6⊂ C. Thus, (x − , x + ) cannot be a subset of C. Therefore, there is no
 > 0 such that (x − , x + ) ⊂ C, so C has no interior point and C is nowhere dense. 

28. Let F be increasing and right continuous, and let µF be the associated measure.
Then
µF ({a}) = F (a) − F (a−),
µF ([a, b)) = F (b−) − F (a−),
µF ([a, b]) = F (b) − F (a−),
and µF ((a, b)) = F (b−) − F (a)
Proof. .
Due to the continuity from above of µF , for any a, b ∈ R where a ≤ b, observe that
!
\ 1 1
µF ([a, b]) = µF (a − , b] = lim µF ((a − , b])
n∈N
n n→∞ n
1
= lim (F (b) − F (a − ))
n→∞ n
1
= F (b) − lim F (a − )
n→∞ n
= F (b) − F (a−)
Also, observe that
µF ({a}) = µF ([a, a]) = F (a) − F (a−)

Due to continuity from below of µF , observe that


!
[ 1 1
µF ((a, b)) = µF (a, b − ] = lim µF ((a, b − ])
n∈N
n n→∞ n
1
= lim (F (b − ) − F (a))
n→∞ n
1
= ( lim F (b − )) − F (a)
n→∞ n
= F (b−) − F (a)
11

Lastly, due to continuity from below of µF and previous consequence,


!
[ 1 1
µF ([a, b)) = µF [a, b − ] = lim µF ([a, b − ])
n∈N
n n→∞ n
1
= lim (F (b − ) − F (a−))
n→∞
 n 
1
= lim (F (b − )) − F (a−)
n→∞ n
= F (b−) − F (a−)


29. Let E be a Lebesgue measurable set. H


a. If E ⊂ N is the nonmeasurable set described in 1.1, then m(E)=0.
b. HIf m(E) >S0, then E contains a nonmeasurable set. (It suffices to assume E ⊂ [0, 1]. In the notation
of 1.1, E = r∈R (E ∩ Nr ))
Proof. . H
Let E be a Lebesgue measurable set and N be the non measurable set in 1.1.
Solution for a.
Let E ⊂ N . Define E + a = {x + a|x ∈ E}.
By way of contradiction, assume that m(E) > 0. Note that for any r ∈ Q ∩ [0, 1], m(E) = m(E + r) ,
E + r ⊂ [0, 2] and E + r ∩ E + q = ∅ if r 6= q where q ∈ Q ∩ [0, 1].
Observe that
 
[
∞ = Σr∈Q∩[0,1] m(E + r) = m  (E + r) ≤ m([0, 2]) = 2
r∈Q∩[0,1]

Since ∞ ≤ 2 is impossible, it is contradiction. Thus, since E is Lebesgue measurable, the only possible
situation is when m(E) = 0.

Solution for b.
Note that, since
0 < m(E) = Σn∈Z m(E ∩ [n, n + 1])
∃n ∈ Z such that m(E ∩ [n, n + 1]) > 0.
Let E0 = {x − n : x ∈ E ∩ [n, n + 1]}, then E0 ⊂ [0, 1]. Since Lebesgue measure is translation invariant,
it is enough to prove it by assuming that E ⊂ [0, 1].

Supposing E ⊂ [0, 1], if E ∩ Nr is measurable ∀r ∈ Q ∩ [0, 1], then, by the result of a,


m(E) = Σr∈Q∩[0,1] m(E ∩ Nr ) = 0
S
We can do it because each Nr are disjoint and r∈Q∩[0,1] Nr = [0, 1].
However, it is contradiction because we have a condition m(E) > 0, so if m(E) > 0, then E contains
nonmeasurable set. 

30.
Proof. 

31.
12

Proof. 

32.
Proof. 

33.
Proof. 

34.
Proof. 

35.
Proof. 

36.
Proof. 

37.
Proof. 

38.
Proof. 

39.
Proof. 

40.
Proof. 

41.
Proof. 

42.
Proof. 

43.
13

Proof. 

44.
Proof. 

45.
Proof. 

46.
Proof. 

47.
Proof. 

48.
Proof. 

49.
Proof. 

50.
Proof. 

51.
Proof. 

52.
Proof. 

53.
Proof. 

54.
Proof. 
14

55.
Proof. 

56.
Proof. 

57.
Proof. 

58.
Proof. 

You might also like